Comparaison

Bonsoir
Si $\epsilon^2 \omega \to 0$ est-ce que $\epsilon^2 \omega < (\epsilon^2 \omega)^J$ où $J \in \N$ et $\omega >0$ et $\epsilon$ petit ?
Merci par avance pour toute réponse.

Réponses

  • Ça ne veut pas dire grand-chose, mais si l'on doit comprendre que $0 \leq \epsilon^2 \omega < 1$ alors la réponse est bien évidemment oui.
  • Tend vers $0$ quand qui que quoi dont où tend vers qui que quoi dont où?
  • oui Poirot, $0 <\epsilon^2 \omega <1$ alors on a bien $\epsilon^2 \omega < (\epsilon^2 \omega)^J$
  • Vu que $\epsilon$ est petit, je pense que c'est lui qui rend vers $0$.
  • Bonjour,

    La réponse est oui sauf que avec l’inégalité stricte c’est faux pour $J=1$.
  • Ma deuxième question est si $0< \epsilon^2 \omega < 1$ pourquoi est-ce que si
    $R \leq O(\epsilon \sqrt{\omega})+ O(\epsilon \epsilon\sqrt{\omega}) + O(\sqrt{\omega}(\epsilon^2 \omega)^J)$ alors s'il existe $J \in \N$ tel que $\sqrt{\omega}(\epsilon^2 \omega)^J \to 0$ on a $R < O(\epsilon \sqrt{\omega}+ \sqrt{\omega}(\epsilon^2 \omega)^J)$ ? Où $O$ est le reste de Lambdau.
  • Quand tu mets des limites, spécifie qui tend vers quoi.
  • @Algèbre: $\epsilon^2 \omega <<1$ et $\epsilon \to 0$. Pourquoi est-ce que si
    $R \leq O(\epsilon \sqrt{\omega})+ O(\epsilon \epsilon\sqrt{\omega}) + O(\sqrt{\omega}(\epsilon^2 \omega)^J)$ alors s'il existe $J \in \N$ tel que $\sqrt{\omega}(\epsilon^2 \omega)^J \to 0$ on a $R < O(\epsilon \sqrt{\omega}+ \sqrt{\omega}(\epsilon^2 \omega)^J)$ ? Où $O$ est le reste de Lambdau.
  • $O(\epsilon^2) = O(\epsilon)$ quand $\epsilon \to 0$.
  • Un peu ambigu, cette égalité pas symétrique... Si une fonction est un $O(\epsilon^2)$, alors c'est a fortiori un $O(\epsilon)$.
  • Le trucs c'est que pour tout $J$ entier non nul $\sqrt{\omega}(\epsilon^2 \omega)^J \to 0$ quand $\epsilon \to 0$. On ne se sert pas de cette hypothèse toujours vrai, le message de Poirot suffit.
  • Si $\epsilon^2 \omega <<1$ alors est-ce qu'il existe $J \in \N$ tel que $\sqrt{\omega}(\epsilon^2 \omega)^J \to 0$? le problème est que dans ce que je lis, il n'est pas dit sur quoi on passe à la limite. Si c'était quand $\epsilon \to 0$ alors c'est vrai pour tout $J \in \N$ et on n'aurait pas dit il existe $J \in \N$.
  • Sans avoir plus d'information sur les dépendances des variables entre elles on ne peut rien dire. Le plus plausible est que $\epsilon$ et $\omega$ sont fonctions d'une même variable que l'on fait tendre vers quelque chose. Pour répondre à ta question, il faudrait connaître la "vitesse" de $\omega$ par rapport à celle de $\epsilon$.
  • On sait seulement que $\omega > 0$ et $\epsilon$ est petit et destiné à tendre vers 0.Ces informations suffisent?
  • Aussi pour j=0?
    Le 😄 Farceur


  • Non gebrane, pour tout $J \geq 1$. Mais sur quoi on passe à la limite pour dire qu'il existe un $J \in \N$ tel que $(\epsilon^2 \omega)^J \to 0$ et pas quelque soit $J \in \N$? En sachant que $\omega > 0$ et $\epsilon$ est petit est destiné à tendre vers 0 et $\epsilon^2 \omega <<1$.
  • De coutume, c'est le $\epsilon$ qui tend vers 0. Je ne peux pas t'aider car je ne sais pas comment dépend w de $\epsilon$. Pour ne pas tromper le monde note w par $w(\epsilon)$
    Le 😄 Farceur


  • Je n'ai jamais dit que $\omega$ dépend de $\epsilon$. $\omega$ ne dépend pas de $\epsilon$!
  • toupii écrivait:
    > oui Poirot, $0 <\epsilon^2 \omega <1$ alors on a bien $\epsilon^2 \omega < (\epsilon^2 \omega)^J$

    De quoi de quoi ? C'est bien sûr faux pour $J\geq 1$.
  • GaBuZoMeu tu peux m'aider? on a $\epsilon^2 \omega <<1$ et il est dit que s'il existe existe $J \in \N$ tel que $(\epsilon^2 \omega)^J \to 0$ alors $R \leq O(\epsilon \sqrt{\omega})+ O(\epsilon^2 \sqrt{\omega})+ O(\sqrt{\omega}(\epsilon^2 \omega)^J)) $ implique que $R \leq O(\epsilon\sqrt{\omega}+ \sqrt{\omega}(\epsilon^2 \omega)^J)$
    a- Pourquoi un tel $J$ existe? et $(\epsilon^2 \omega)^J \to 0$ c'est quand $\epsilon \to 0$? Car ce n'est pas préciser dans le document. Mais si c'est quand $\epsilon \to $ alors c'est quelque soit $J \in \N$:-S
    b- comment justifier la dernière implication?
  • C'est trop incohérent pour que je puisse t'aider. Je ne suis pas sûr que tu interprètes correctement le document.
  • Si tes parametres sont indépendants, ta question devient idiote. Touves-tu vraiment des difficultés à calculer la limite de $(x^2y)^p$ (p>0) lorsque x ou y tend vers 0
    Le 😄 Farceur


Connectez-vous ou Inscrivez-vous pour répondre.